Đến nội dung

Hoang Tung 126 nội dung

Có 1000 mục bởi Hoang Tung 126 (Tìm giới hạn từ 30-04-2020)



Sắp theo                Sắp xếp  

#570496 Min $P=\sum{\frac{\sqrt{ab}}...

Đã gửi bởi Hoang Tung 126 on 08-07-2015 - 11:00 trong Bất đẳng thức và cực trị

Cho ba số $a,b,c$ không âm, tìm GTNN của

$P=\frac{\sqrt{ab}}{c+3\sqrt{ab}}+\frac{\sqrt{bc}}{a+3\sqrt{bc}}+\frac{\sqrt{ca}}{b+3\sqrt{ca}}$

Theo Bunhiacopxki và Cosi ta có :

 

  $\sum \frac{c}{c+3\sqrt{ab}}\geq \sum \frac{c}{c+3.\frac{a+b}{2}}=2\sum \frac{c}{2c+3a+3b}=2\sum \frac{c^2}{2c^2+3ac+3bc}\geq 2.\frac{(\sum c)^2}{2\sum c^2+6\sum ab}=\frac{2(\sum c)^2}{2(\sum c)^2+2\sum ab}\geq \frac{2(\sum c)^2}{2(\sum c)^2+\frac{2(\sum c)^2}{3}}=\frac{6(\sum c)^2}{8(\sum c)^2}=\frac{6}{8}=\frac{3}{4}= > \sum \frac{c}{c+3\sqrt{ab}}\geq \frac{3}{4}= > \sum (1-\frac{c}{c+3\sqrt{ab}})\leq \frac{9}{4}= > P=\sum \frac{\sqrt{ab}}{c+3\sqrt{ab}}\leq \frac{3}{4}$

 

 




#570487 $\sum \frac{a+b}{1-ab}\leq 3(\su...

Đã gửi bởi Hoang Tung 126 on 08-07-2015 - 10:28 trong Bất đẳng thức - Cực trị

  Bài toán : Cho các số thực dương $a,b,c> 0$ thỏa mãn $a^2+b^2+c^2=1$. CMR :

 

           $\frac{a+b}{1-ab}+\frac{b+c}{1-bc}+\frac{c+a}{1-ac}\leq 3(a+b+c)$

 

 




#570479 $\prod (a^{2015}-a^{2013}+3)\geq 9(\s...

Đã gửi bởi Hoang Tung 126 on 08-07-2015 - 10:02 trong Bất đẳng thức - Cực trị

   Bài toán : Cho các số thực dương $a,b,c> 0$ . CMR:

 

     $(a^{2015}-a^{2013}+3)(b^{2015}-b^{2013}+3)(c^{2015}-c^{2013}+3)\geq 9(ab+bc+ac)$

 

 




#570418 $\left\{\begin{matrix} f(x)-2g(x)=g(y)+4y...

Đã gửi bởi Hoang Tung 126 on 07-07-2015 - 21:18 trong Phương trình hàm

   Bài toán : Tìm tất cả các hàm $f:R\rightarrow R, g:R\rightarrow R$ với mọi số thực $x,y$ thỏa mãn :

 

     $\left\{\begin{matrix} f(x)-2g(x)=g(y)+4y & \\ f(x)g(x)\leq 2015x^2 & \end{matrix}\right.$

 

 




#570300 Bánh canh chém gió về kì thi IMO 2015

Đã gửi bởi Hoang Tung 126 on 06-07-2015 - 22:36 trong Thi HSG Quốc gia và Quốc tế

Anh Huy hoàng được vào cả trận chung két Olympia nữa 




#570293 CMR: $\prod (a-1+\frac{1}{b})\leq 1...

Đã gửi bởi Hoang Tung 126 on 06-07-2015 - 21:58 trong Bất đẳng thức - Cực trị

IMO 2000 đó ạ

 Thảo nào nhìn qua quen quen




#570270 $(a+b^2)(b+c^2)(c+a^2)\leq 13$

Đã gửi bởi Hoang Tung 126 on 06-07-2015 - 20:45 trong Bất đẳng thức và cực trị

Em nghĩ là với $a,b,c$ dương thì biểu thức chỉ đạt MAX là 8 thôi 

 Thế dấu =xảy ra khi nào




#570267 $(a+b^2)(b+c^2)(c+a^2)\leq 13$

Đã gửi bởi Hoang Tung 126 on 06-07-2015 - 20:32 trong Bất đẳng thức và cực trị

Bài này không tồn tại dấu $"="$ đâu a!  :wacko: 

Vậy à ,thảo nào tìm mãi ko ra ,mà e đã có ý tưởng bài này chưa




#570265 Topic tổng hợp một số bất đẳng thức trong kì thi MO các nước

Đã gửi bởi Hoang Tung 126 on 06-07-2015 - 20:28 trong Bất đẳng thức - Cực trị

 Dạo này TOPIC ít người tham gia vậy ,vẫn những bài toán từ hôm trước




#570264 $(a+b^2)(b+c^2)(c+a^2)\leq 13$

Đã gửi bởi Hoang Tung 126 on 06-07-2015 - 20:25 trong Bất đẳng thức và cực trị

Bài này dấu = xảy ra thực sự rất khó đoán




#570263 CMR: $\prod (a-1+\frac{1}{b})\leq 1...

Đã gửi bởi Hoang Tung 126 on 06-07-2015 - 20:24 trong Bất đẳng thức - Cực trị

Bài này đã nằm trong đề thi Toán Quốc Tế thì phải




#570148 $\sum \frac{a}{a+b+7c}+\frac{2(...

Đã gửi bởi Hoang Tung 126 on 06-07-2015 - 08:20 trong Bất đẳng thức - Cực trị

  Bài toán : Cho các số thực dương $a,b,c> 0$ .CMR:

 

  $\frac{a}{a+b+7c}+\frac{b}{b+c+7a}+\frac{c}{c+a+7b}+\frac{2(ab+bc+ac)}{3(a^2+b^2+c^2)}\leq 1$

 

 




#570146 $I=\int_{1}^{2}\dfrac{dx}{x...

Đã gửi bởi Hoang Tung 126 on 06-07-2015 - 07:53 trong Tích phân - Nguyên hàm

Tính tích phân sau :

$$I=\int_{1}^{2}\dfrac{dx}{x(x^3+1)}$$

Ta có : $I=\int_{1}^{2}\frac{dx}{x(x^3+1)}=\int_{1}^{2}\frac{x^2}{x^3(x^3+1)}dx$

 

  Đặt $x^3+1=t= > dt=3x^2dx$

 

Chuyển cận $\left\{\begin{matrix} x=1= > t=2 & \\ x=2= > t=9 & \end{matrix}\right.$ 

 

Áp dụng công thức $\int \frac{1}{x}=ln\left | x \right |$

 

 $= > I=\frac{1}{3}\int_{1}^{2}\frac{3x^2}{x^3(x^3+1)}dx=\frac{1}{3}\int_{2}^{9}\frac{dt}{t(t-1)}=\frac{1}{3}\int_{2}^{9}(\frac{1}{t-1}-\frac{1}{t})dt$
$=\frac{1}{3}\int_{2}^{9}\frac{1}{t-1}dt-\frac{1}{3}\int_{2}^{9}\frac{1}{t}dt$
$=\frac{1}{3}(ln8-ln1-ln9+ln2)=\frac{ln\frac{16}{9}}{3}$
$= > I=\frac{ln\frac{16}{9}}{3}$




#570144 $\sqrt{x^{3}y+y^{3}z+z^{3}x...

Đã gửi bởi Hoang Tung 126 on 06-07-2015 - 07:41 trong Bất đẳng thức và cực trị

Cho $\left\{\begin{matrix}x\geq 0,y\geq 0,z\geq 0 & & \\ & & \end{matrix}\right.x+y+z=2$

 

Chứng minh rằng : $\sqrt{x^{3}y+y^{3}z+z^{3}x} + \sqrt{xy^{3}+yz^{3}+zx^{3}}\leq 2$

Theo Bunhiacopxki ta có :

 

 $\sqrt{x^3y+y^3z+z^3x}+\sqrt{xy^3+yz^3+zx^3}\leq \sqrt{2(x^3y+xy^3+y^3z+yz^3+zx^3+xz^3)}\leq \sqrt{2(xy+yz+xz)(x^2+y^2+z^2)}\leq \frac{x^2+y^2+z^2+2(xy+yz+xz)}{2}=\frac{(x+y+z)^2}{2}=\frac{2^2}{2}=2$

 

  Dấu = xảy ra khi  có 2 số =1 ,1 số =0




#569950 $\frac{f(x)+f(y)}{\sqrt{f(xy)}}=...

Đã gửi bởi Hoang Tung 126 on 04-07-2015 - 21:19 trong Phương trình hàm

 Tìm tất cả các hàm số $f:(0;+\infty)\to (0;+\infty)$ sao cho $$\frac{f(x)+f(y)}{\sqrt{f(xy)}}=\frac{x+y}{\sqrt{xy}}\forall x,y\in (0;+\infty).$$

Đề thi chọn HSG QG Tỉnh Quảng Trị năm học 2014 - -2015

-Chọn $x=y=1= > \frac{f(1)+f(1)}{\sqrt{f(1)}}=\frac{1+1}{\sqrt{1}}= > \frac{2f(1)}{\sqrt{f(1)}}=2= > f(1)=\sqrt{f(1)}= > f(1)=1$

   (Do hàm $f:(0,+\infty )\rightarrow (0,+\infty )$  nên $f(1)$ khác $0$)

 

- Chọn $y=1$ và áp dụng $f(1)=1$

 

 $= > \frac{f(x)+f(1)}{\sqrt{f(x)}}=\frac{x+1}{\sqrt{x}}= > \frac{f(x)+1}{\sqrt{f(x)}}=\frac{x+1}{\sqrt{x}}= > \sqrt{f(x)}-\sqrt{x}=\frac{1}{\sqrt{x}}-\frac{1}{\sqrt{f(x)}}= > (\sqrt{f(x)}-\sqrt{x})(1-\frac{1}{\sqrt{xf(x)}})=0$

 

 Do đó $f(x)=x$ hoặc $f(x)=\frac{1}{x}$ .Thử lại ta thấy thỏa mãn bài toán




#569929 min $P=a^5b+b^5a+\dfrac{6}{a^2+b^2}-3(a+b)$

Đã gửi bởi Hoang Tung 126 on 04-07-2015 - 19:41 trong Bất đẳng thức và cực trị

Cho các số thực $a,b$ thỏa mãn $a,b\in \left[\dfrac{1}{2};1\right]$. Tìm giá trị nhỏ nhất của biểu thức $P=a^5b+b^5a+\dfrac{6}{a^2+b^2}-3(a+b)$

- Do $a,b$ thuộc đoạn $\left [ \frac{1}{2},1 \right ]= > 1-a\geq 0,1-b\geq 0= > (1-a)(1-b)\geq 0= > ab\geq a+b-1$

 

Do $a,b\geq \frac{1}{2}= > a+b-1\geq \frac{1}{2}+\frac{1}{2}-1=0= > a+b-1\geq 0$

 

  Áp dụng bđt $m^{4}+n^{4}\geq \frac{(m+n)^4}{8}$

 

Từ đó ta có:

 

   $P=a^5b+ab^5+\frac{6}{a^2+b^2}-3(a+b)=ab(a^4+b^4)+\frac{6}{(a+b)^2-2ab}-3(a+b)$
$\geq (a+b-1).\frac{(a+b)^4}{8}+\frac{6}{(a+b)^2-2(a+b-1)}-3(a+b)$
$=\frac{(a+b)^5-(a+b)^4}{8}+\frac{6}{(a+b)^2-2(a+b)+2}-3(a+b)$
$= > P\geq \frac{t^5-t^4}{8}+\frac{6}{t^2-2t+2}-3t$

 

 (với $t=a+b$)

 

 Ta chứng minh $P\geq -1< = > \frac{t^5-t^4}{8}+\frac{6}{t^2-2t+2}-3t\geq -1< = > (t-2)^2(t^5+t^4+4t^3+10t^2+16)\geq 0$ 

    (Điều này luôn đúng)

 

Do đó  $P\geq -1= > P_{min}=-1< = > (a-1)(b-1)=0,a+b=2< = > a=b=1$




#569915 $\frac{l_{a}+l_{b}+l_{c}}...

Đã gửi bởi Hoang Tung 126 on 04-07-2015 - 18:07 trong Hình học

  Bài toán : Cho tam giác $ABC$ có 3 góc là $A,B,C$. Gọi $l_{a},l_{b},l_{c}$ theo thứ tự là 3 đường phân giác trong xuất phát từ 3 đỉnh của tam giác . Gọi $R$ là bán kính đường tròn ngoại tiếp tam giác .CMR:

 

       $\frac{l_{a}+l_{b}+l_{c}}{R}\leq 2(cos\frac{A}{2}cos\frac{B}{2}+cos\frac{B}{2}cos\frac{C}{2}+cos\frac{C}{2}cos\frac{A}{2})$

 

 




#569659 Bánh canh chém gió về kì thi IMO 2015

Đã gửi bởi Hoang Tung 126 on 03-07-2015 - 15:28 trong Thi HSG Quốc gia và Quốc tế

 Mong năm nay anh Hoàn sẽ làm quả HCV thứ 2 và là người thứ 7 hai lần giành vàng liên tiếp




#569657 $\sum \frac{a}{b^3+ab}\geq 3$

Đã gửi bởi Hoang Tung 126 on 03-07-2015 - 15:19 trong Bất đẳng thức - Cực trị

1) Cho $a,b,c$ thực dương thỏa $a+b+c=1$. Chứng minh

$\sum \sqrt{\frac{a^2+2ab}{b^2+2c^2}}\geq \frac{1}{a^2+b^2+c^2}$

2) Cho $a,b,c$ thực dương thỏa $a+b+c=3$. Chứng minh

$\sum \frac{a}{b^3+ab}\geq 3$

Bài 2: ta có :

 

 $\sum \frac{a}{b^3+ab}=\sum \frac{a}{b(a+b^2)}=\sum \frac{a+b^2-b^2}{b(a+b^2)}=\sum \frac{1}{b}-\sum \frac{b}{a+b^2}\geq \sum \frac{1}{b}-\sum \frac{b}{2\sqrt{ab^2}}=\sum \frac{1}{b}-\frac{1}{2}\sum \frac{1}{\sqrt{a}}\geq \sum \frac{1}{b}-\frac{1}{2}\sum (\frac{\frac{1}{a}+1}{2})=\sum \frac{1}{b}-\frac{1}{4}\sum \frac{1}{a}-\frac{3}{4}=\frac{3}{4}(\sum \frac{1}{a}-1)\geq \frac{3}{4}(\frac{9}{\sum a}-1)=\frac{3}{4}(\frac{9}{3}-1)=\frac{3}{2}$




#569656 $\sum \frac{a}{b^3+ab}\geq 3$

Đã gửi bởi Hoang Tung 126 on 03-07-2015 - 15:15 trong Bất đẳng thức - Cực trị

1) Cho $a,b,c$ thực dương thỏa $a+b+c=1$. Chứng minh

$\sum \sqrt{\frac{a^2+2ab}{b^2+2c^2}}\geq \frac{1}{a^2+b^2+c^2}$

2) Cho $a,b,c$ thực dương thỏa $a+b+c=3$. Chứng minh

$\sum \frac{a}{b^3+ab}\geq 3$

Bài 1: Ta có : 

 

 $\sum \sqrt{\frac{a^2+2ab}{b^2+2c^2}}=\sum \frac{a^2+2ab}{\sqrt{(b^2+2c^2)(a^2+2ab)}}\geq \sum \frac{a^2+2ab}{\frac{a^2+2ab+b^2+2c^2}{2}}=2\sum \frac{a^2+2ab}{(a+b)^2+2c^2}\geq 2\sum \frac{a^2+2ab}{2(a^2+b^2)+2c^2}=\sum \frac{a^2+2ab}{a^2+b^2+c^2}=\frac{(\sum a)^2}{\sum a^2}=\frac{1}{\sum a^2}$




#569652 $$\sum \frac{a^2+b^2}{a+b}\geq...

Đã gửi bởi Hoang Tung 126 on 03-07-2015 - 15:02 trong Bất đẳng thức - Cực trị

Bài toán:

Chứng minh rằng với mọi số thực dương  $a,b,c$  ta luôn có:

$$\sum \frac{a^2+b^2}{a+b}\geq \sqrt{3(a^2+b^2+c^2)+2abc\sum \frac{b+c-2a}{a(b+c)}}$$

 Ta có :$\sum \frac{b+c-2a}{a(b+c)}=\sum \frac{b-a}{a(b+c)}+\sum \frac{c-a}{a(b+c)}=\sum \frac{b-a}{a(b+c)}-\sum \frac{b-a}{b(a+c)}=\sum (b-a)(\frac{1}{a(b+c)}-\frac{1}{b(a+c)})=\sum (b-a)(\frac{c(b-a)}{ab(b+c)(a+c)})=\sum \frac{c(a-b)^2}{b(b+c)(a+c)}= > 2abc\sum \frac{b+c-2a}{a(b+c)}=2\sum \frac{c^2(a-b)^2}{(b+c)(a+c)}$

 

 Do đó $\sum \frac{a^2+b^2}{a+b}\geq \sqrt{3(\sum a^2)+2abc\sum \frac{b+c-2a}{a(b+c)}}$

$= > (\sum \frac{a^2+b^2}{a+b})^2\geq 3\sum a^2+2\sum \frac{c^2(a-b)^2}{(c+a)(c+b)}$

$< = > \sum (\frac{a^2+b^2}{a+b})^2+2\sum \frac{(a^2+b^2)(a^2+c^2)}{(a+b)(a+c)}\geq 3\sum a^2+2\sum \frac{c^2(a-b)^2}{(c+a)(c+b)}$

$< = > \sum (\frac{(a+b)^2-2ab}{a+b})^2+2\sum \frac{(a^2+b^2)(a^2+c^2)}{(a+b)(a+c)}\geq 3\sum a^2$
$+2\sum \frac{c^2(a-b)^2}{(a+c)(b+c)}$

$< = > \sum \frac{(a+b)^4}{(a+b)^2}-4\sum \frac{ab(a+b)}{a+b}+\sum \frac{(2ab)^2}{(a+b)^2}+2\sum \frac{(a^2+c^2)(b^2+c^2)}{(a+c)(b+c)}\geq 3\sum a^2$
$+2\sum \frac{c^2(a-b)^2}{(a+c)(b+c)}$

$< = > \sum (a+b)^2-4\sum ab+\sum \frac{(2ab)^2}{(a+b)^2}+2\sum \frac{(a^2+c^2)(b^2+c^2)}{(a+c)(b+c)}\geq 3\sum a^2$
$+2\sum \frac{c^2(a-b)^2}{(c+a)(c+b)}$

$< = > 2\sum \frac{(a^2+c^2)(b^2+c^2)}{(a+c)(b+c)}-2\sum \frac{c^2(a-b)^2}{(c+a)(c+b)}+\sum \frac{(2ab)^2}{(a+b)^2}\geq 3\sum a^2-\sum (a+b)^2+4\sum ab$

$< = > 2\sum \frac{(a^2+c^2)(b^2+c^2)-c^2(a-b)^2}{(a+c)(b+c)}+\sum (\frac{2ab}{a+b})^2\geq \sum a^2+2\sum ab$

$< = > 2\sum \frac{(c^2+ab)^2}{(c+a)(c+b)}+\sum (\frac{2ab}{a+b})^2\geq (\sum a)^2$  (1)

 

  Nhưng theo BDT Bunhiacopxki ta có :

 

   $2\sum \frac{(c^2+ab)^2}{(c+a)(c+b)}+\sum (\frac{2ab}{a+b})^2$

$=\sum \frac{(c^2+ab)^2}{(c+a)(c+b)}+\sum \frac{(c^2+ab)^2}{(c+a)(c+b)}+\sum \frac{(2ab)^2}{(a+b)^2}\geq \frac{\left [ \sum (c^2+ab)+\sum (c^2+ab)+2\sum ab \right ]^2}{2\sum (c+a)(c+b)+\sum (a+b)^2}$

$=\frac{(2\sum c^2+4\sum ab)^2}{4\sum c^2+8\sum ab}=\frac{4(\sum a)^4}{4(\sum a)^2}=(\sum a)^2$

$= > 2\sum \frac{(c^2+ab)^2}{(c+a)(c+b)}+\sum (\frac{2ab}{a+b})^2\geq (\sum a)^2$  

 

    Do đó BDT (1) đúng và ta có ĐPCM .Dấu = xảy ra tại $a=b=c$




#569537 $\sum (\frac{a}{a+b})^2+3\geq \f...

Đã gửi bởi Hoang Tung 126 on 02-07-2015 - 21:53 trong Bất đẳng thức và cực trị

Ý thầy là tìm $k$ tốt nhất để

$\sum k(\dfrac{a}{a+b})^2 \geq \dfrac{5}{2}(\sum \dfrac{a}{a+b})+\dfrac{3k}{4}-\dfrac{15}{4}.$

vâng ,để em xem lại 




#569531 $\sum (\frac{a}{a+b})^2+3\geq \f...

Đã gửi bởi Hoang Tung 126 on 02-07-2015 - 21:23 trong Bất đẳng thức và cực trị

  Ngoài ra ta cũng có thể thay số $\frac{5}{2}=k$ và tìm hằng số $k$ tốt nhất thỏa mãn bài toán.

 

Thật vậy ,BĐT $< = > \sum (\frac{a}{a+b})^2+3\geq k(\sum \frac{a}{a+b})< = > \sum \frac{1}{(1+\frac{b}{a})^2}+3\geq k(\sum \frac{1}{1+\frac{b}{a}})$

 

  Đặt $\frac{b}{a}=x,\frac{c}{b}=y,\frac{a}{c}=z= > xyz=1$

 

BĐT $< = > \sum \frac{1}{(x+1)^2}+3\geq k(\sum \frac{1}{x+1})$

 

-Chọn $x=y=m,z=\frac{1}{m^2}(m> 0)$

 

BĐT $< = > \frac{1}{(m+1)^2}+\frac{1}{(m+1)^2}+\frac{1}{(1+\frac{1}{m^2})^2}+3\geq k(\frac{1}{m+1}+\frac{1}{m+1}+\frac{1}{1+\frac{1}{m^2}})< = > \frac{2}{(m+1)^2}+\frac{m^2}{(m+2)^2}+3\geq k(\frac{2}{m+1}+\frac{m^2}{m^2+1})= > \frac{m^6+2m^5+3m^4+4m^2+2+3(m^2+2m+1)(m^4+2m^2+1)}{(m+1)^2(m^2+1)^2}\geq k(\frac{m^3+3m^2+2}{(m+1)(m^2+1)})$

$= > k\leq \frac{4m^6+8m^5+12m^4+12m^3+13m^2+6m+5}{(m+1)(m^2+1)(m^3+3m^2+2)}$ (1)

 

 - Cho $x=y\rightarrow z= > m\rightarrow \frac{1}{m^2}= >m\rightarrow 1$

 

Từ (1) $= > k\leq \lim_{m\rightarrow 1}\frac{4m^6+8m^5+12m^4+12m^3+13m^2+6m+5}{(m+1)(m^2+1)(m^3+3m^2+2)}=\frac{60}{4.6}=\frac{5}{2}= > k\leq \frac{5}{2}= > k_{max}=\frac{5}{2}$




#569513 $\sum (\frac{a}{a+b})^2+3\geq \f...

Đã gửi bởi Hoang Tung 126 on 02-07-2015 - 20:08 trong Bất đẳng thức và cực trị

Một bài toán hay và có nhiều cách giải ( Cách xấu nhất là quy đồng và dùng BW)

 Đúng vậy ,bài này có nhiều cách giải ,nhưng cách quy đồng là tự nhiên nhất




#569492 $\sum (\frac{a}{a+b})^2+3\geq \f...

Đã gửi bởi Hoang Tung 126 on 02-07-2015 - 17:09 trong Bất đẳng thức và cực trị

Ngoài lề: Các anh cho em hỏi hạn nộp bài là như thế nào ạ :D

Ví dụ bài này có hạn nộp là 30/6 chẳng hạn